Find the value of x. If necessary, round to the nearest tenth

Find The Value Of X. If Necessary, Round To The Nearest Tenth

Answers

Answer 1

Answer:

The picture in the attached figure. we know that Area of the triangle is equal to A= ( b×h ) /2 in this problem

A=47 in² b=x h=x

so 47 = X * X / 2

x² = 94 [tex]x = \sqrt{94} [/tex]

[tex]x = 9.69in [/tex]

[tex]x = 9.7in[/tex]

Therefore

the answer is

[tex]x = 9.7in[/tex]

And please follow me...

Related Questions

If using the method of completing the square to solve the quadratic equation
x^2 + 8x – 4 = 0, which number would have to be added to "complete the square"?

Answers

Step-by-step explanation:

everything can be found in the picture

A grocery store chain uses loyalty cards that offer discounts on sale attheir store. The probability that a randomly chosen shopper has a loyaltycard is 0.8. A group of shoppers have been selected at random to completea customer satisfaction survey. Let Xi be a random variable that equals 1 when a customer has a loyalty card and 0 otherwise.

Required:
a. Construct a joint probability table for the random variables X, and X.
b. Define the event that the first four customers sampled have loyalty cards in terms of the random variables X, X2, X3, and Xd. Then compute the probability of this event.

Answers

The answer is a cause

plz it's do to day HELP PLZ​

Answers

Answer:

9 units^3

Step-by-step explanation:

V = l*w*h

   = 1 1/2 * 4 * 1 1/2

Change to improper fractions

  = 3/2 * 4 * 3/2

   = 36/4

   = 9 units^3

3.
4.
Every weekend Felicia is paid $30 to do the gardening for her
grandparents. She is saving her money to buy a new bike that
costs $189. So far, she has saved $60.
What is the minimum number of weekends she'll have to
continue to do the gardening in order to have the $189 she
needs for the bike?

Answers

Answer:

5 weekends

Step-by-step explanation:

Felicia already has 60$ of her goal of 189$ so 189-60=129

129/30=4.3

Felicia would need to work exactly 4.3 weekends to make up her goal, but you cant split a weekend into decimals, so we round up to the next whole number which is 5.

Answer:

she needs to work a minimum of 5 weekends

Step-by-step explanation:

60 + 30w ≥ 189

30w ≥ 129

w ≥ 4.3

Let the sequence be 1, 3, 5, 7, 9……… then this sequence is

Answers

the answer is 11 because 2 multiple are increasing

Answer:

An arithmetic sequence, The difference in any term with the previous term is same.

Step-by-step explanation:

See, ah this is a little bit easy question so let’s start solving it -:

Firstly, um see the difference between the given numbers-: 1, 3, 5, 7,9 _?

Between 1 and 3 the difference is 2

Between 5 and 3 the difference is 2

Between 7 and 5 the difference is 2

Between 9 and 7 the difference is 2

So, here we see the difference among all is same. So. it is an arithmetic sequence.

So, let's   find the fourth term. We can either do it by arithmetic progression or  by simply adding 2 to 7. I.ll first show you by doing A.P. then we will check the answer by adding 2 to 7.

So, as we know the basic formula of A.P. which is -: a1+(n-1)d

Here, a1=1, n=5, d=2

Let’s put the values in the formula-: 1+(5–1)2 = 1+(4)2 = 1+8 = 9

Now let’s check it. As the difference between each digit is same so definately the difference between 9 and the unknown number is 2.

Then, the unknown no. is 9+2 = 11

Hence, the answer is 11

csc theta * cos^2 theta +sin theta= csc theta

Answers

Step-by-step explanation:

[tex] \csc \theta \cos^{2}\theta + \sin \theta = \csc \theta(1 - \sin^{2}\theta) + \sin \theta[/tex]

[tex] = (\csc \theta - \sin \theta) + \sin \theta[/tex]

[tex] = \csc \theta[/tex]

100 points if u get it right!!!







Answers

Answer:

To be honest I think its C but it may be A

Step-by-step explanation:

C? i think most likely

Find the equal measures of 1. 28 m = ________ km 2. 3899 ml = ______ l 3. 79 kg = _____ g Show the process of each

Answers

Answer:

1. 0.028 km

2. 3.899 L

3. 79,000 g

Simplify (2 1/6 - 1 2/3) - 2 2/3​

Answers

Answer:

[tex]- 2\frac{1}{6}[/tex]

Step-by-step explanation:

[tex](2\frac{1}{6} -1\frac{2}{3}) - 2 \frac{2}{3}[/tex]

[tex](\frac{13}{6} -\frac{5}{3}) - 2 \frac{2}{3}[/tex]

[tex](\frac{13}{6} -\frac{10}{6}) - 2 \frac{2}{3}[/tex]

[tex]\frac{3}{6} - 2 \frac{2}{3}[/tex]

[tex]\frac{3}{6} - \frac{8}{3}[/tex]

[tex]\frac{3}{6} - \frac{16}{6}[/tex]

[tex]- \frac{13}{6}[/tex]

[tex]- 2\frac{1}{6}[/tex]

Jackson made 36 calls in 2.25 hours. How many calls per hour did he make?

Answers

Answer:

16 calls

Step-by-step explanation:

make an equation based off this ratio -- 36 calls: 2.25 hours and x calls: 1 hour

36/2.25 = x/1

cross multiply to get 36=2.25x

divided by 2.25 on both sides

x=16

Figure ABCDE has vertices A(-2, 3), B(2, 3), C(5,-2), D(0, -4), and E(-2,-2). Plot the points on your own coordinate grid and connect the points in alphabetical order. Decompose Figure ABCDE into rectangles and triangles. Part A: How many triangles and rectangles did you make? (1 point) Part B: Use Figure ABCDE created on your coordinate grid to find the lengths, in units, of Sides AB and AE. (4 points) Part C: What is the area of Figure ABCDE? Show your work. (5 points)​

Answers

9514 1404 393

Answer:

  A: 1 rectangle and 2 triangles (or 1 trapezoid and 1 triangle)

  B: AB = 4; AE = 5

  C: area = 34.5 square units

Step-by-step explanation:

Part A

The figure can be decomposed into a right trapezoid and a triangle. The right trapezoid can be decomposed into a rectangle and a triangle.

easiest decomposition: 1 right trapezoid and 1 triangle

decomposition per instructions: 1 rectangle and 2 triangles.

__

Part B

AB is a horizontal line, so its length is the difference of the x-coordinates of B and A: 2 -(-2) = 4.

AE is a vertical line, so its length is the difference of the y-coordinates of A and E: 3 -(-2) = 5.

  AB = 4, AE = 5

__

Part C

The area of the trapezoid is given by ...

  A = 1/2(b1 +b2)h

The figure shows the base lengths to be 4 and 7, and the height to be 5. The trapezoid's area is ...

  A = 1/2(4+7)(5) = 27.5 . . . . square units

The area of triangle CDE is given by the formula ...

  A = 1/2bh

The figure shows the base length to be 7 and the height to be 2 units. The triangle area is ...

  A = 1/2(7)(2) = 7 . . . . square units

The total area of figure ABCDE is 27.5 +7 = 34.5 square units.

Answer:

Answer:

 A: 1 rectangle and 2 triangles (or 1 trapezoid and 1 triangle)

 B: AB = 4; AE = 5

 C: area = 34.5 square units

Step-by-step explanation:

Part A

The figure can be decomposed into a right trapezoid and a triangle. The right trapezoid can be decomposed into a rectangle and a triangle.

easiest decomposition: 1 right trapezoid and 1 triangle

decomposition per instructions: 1 rectangle and 2 triangles.

__

Part B

AB is a horizontal line, so its length is the difference of the x-coordinates of B and A: 2 -(-2) = 4.

AE is a vertical line, so its length is the difference of the y-coordinates of A and E: 3 -(-2) = 5.

 AB = 4, AE = 5

__

Part C

The area of the trapezoid is given by ...

 A = 1/2(b1 +b2)h

The figure shows the base lengths to be 4 and 7, and the height to be 5. The trapezoid's area is ...

 A = 1/2(4+7)(5) = 27.5 . . . . square units

The area of triangle CDE is given by the formula ...

 A = 1/2bh

The figure shows the base length to be 7 and the height to be 2 units. The triangle area is ...

 A = 1/2(7)(2) = 7 . . . . square units

The total area of figure ABCDE is 27.5 +7 = 34.5 square units.

Step-by-step explanation:

trust meh

Tell wether x and y show a proportional relationship.

x 1 2 3 4
y 4 8 12 16

Answers

Answer:

They do because y is always 4 times of x.

Step-by-step explanation:

There are 27 chocolates in a box, all identically shaped. They’re 10 are filled with nuts, eat with Carmel, and nine are solid chocolate. You randomly select one piece, eat it, and then select a second piece find the probability of selecting two solid chocolate in a row

Answers

There are 27 chocolates. 10 are nuts ( 37%). 9 are chocolates (33%). 8 are caremel (30%). So 37/2 = 16.5% is the answer.

The club with an occupancy of 100 will sell out at $25 per ticket. If the owner decides to increase the price by $5, then 2 less tickets will be sold. Determine the revenue function if x is the number of $5 increases.

Answers

Answer:

100 + (5X) = -2X

Step-by-step explanation:

Since the club with an occupancy of 100 will sell out at $ 25 per ticket, and if the owner decides to increase the price by $ 5, then 2 less tickets will be sold, to determine the revenue function if X is the number of $ 5 increases the following calculation must be performed:

100 + (5X) = -2X

So, for example, if the price were increased by $ 15, the equation would apply as follows:

15/5 = 3

100 + (5x3) = -2 x 3

100 + 15 = -6

115 = -6

What is the value of y in the equation 3y - 8 = 22?
7.3
8
10
14

Answers

The answer to your question is ten

The rate to rent a small moving van is $30 per day, plus $2 per mile. Jada rented a van to drive to her new home. It took
two days, and the total cost was $360. How many miles did she drive?

Answers

150 miles cause 150 x 2 is 300 plus the $30 a day fee

0.36% as a fraction in simplest form

Answers

Answer: 9/2500

Step-by-step explanation:

We start with turning it into a fraction:

0.36/100

Then, we turn every number into a whole number by multiplying by whatever the last decimal place is called (hundreth= x100)

36/10000

Lastly, we simplify as much as we can by finding the least common denominator. In this case, the smallest whole number both 36 and 10000 can be divided by is 4:

(36/4)/(10000/4)

= 9/25000

Indicate the measure of each angle when ∠PTS=128

Answers

9514 1404 393

Answer:

  ∠PTR = 80°

  ∠PTQ = 45°

  ∠QTS = 83°

Step-by-step explanation:

The outer scale is useful for reading the values of angles that start with PT.

  ∡PTR = 80°

  ∡PTQ = 45°

The measure of angle QTS is found from ...

  ∡PTS = ∡PTQ +∡QTS . . . . . angle sum theorem

  ∡QTS = ∡PTS -∡PTQ . . . . . subtract ∡PTQ

  ∡QTS = 128° -45°

  ∡QTS = 83°

A student is taking a multiple-choice exam in which each question has four choices.

Assume that the student has no knowledge of the correct answers to any of the questions.

She has decided on a strategy in which she will place four balls (marked A, B, C, and D) into a box.

She randomly selects one ball for each question and replaces the ball in the box.

The marking on the ball will determine her answer to the question.

There are five multiple choice questions on the exam.

What is the probability that she will get

a. Five questions correct?

b. At least four questions correct?

c. No questions correct?

d. No more than two questions correct?


Answers

Answer:

d no more than two questions correct

Please answer this, I am confused (5^2)(7^2)(3^2)

Answers

Given:

The expression is:

[tex](5^2)(7^2)(3^2)[/tex]

To find:

The value of the given expression.

Solution:

We have,

[tex](5^2)(7^2)(3^2)[/tex]

On simplification, we get

[tex](5^2)(7^2)(3^2)=(25)(49)(9)[/tex]

[tex](5^2)(7^2)(3^2)=11025[/tex]

Therefore, the value of the given expression is 11025.

please help me with this​

Answers

Answer:

28.24

Step-by-step explanation:

25 + 32/10 + 4/100

take lcm of the denominator

lcm = 100

25*100 + 32*10 +4*1/100

2500 + 320 +4/100

2824/100

28.24

Find the value of (1 point)

Answer D .28.24

The ages of each of the basketball players on a team are given.
13, 13, 13, 14, 14, 17, 18, 18
What is the mean age of these players?

Answers

Answer:

15

Step-by-step explanation:

We add up the ages of all the basketball players.

13 + 13 + 13 + 14 + 14 + 17 + 18 + 18 = 120

Now we divide by the number of basketball players.

120 ÷ 8 = 15

The mean age of the basketball players are 15.

Answer:

Mean = 15

Step-by-step explanation:

Mean = sum of observation. ÷ total no. of observation

= 13 + 13 + 13 + 14 + 14 + 17 + 18 + 18 ÷ 8= 120 ÷ 8= 15

WILL MARK BRAINLIEST!!
Which is the graph of g(x)

Answers

Answer:

the second one after first

Step-by-step explanation:

it strictly obeys the rules of boundary

Solve for M and N please help me

Answers

Answer:

M=4   N=6

Hope this helped!

An aeroplane descended 1.4 miles to an elevation of 0.6 miles as shown by the equation below.
Solve the equation to find the elevation of the plane before its descent.
x-1.4 = 0.6
A 0.8
B 1.0
C 2.0
D 1.2
Quick

Answers

Answer:

C 2.0

Step-by-step explanation:

x-1.4 =0.6

add 1.4 to both sides

x=2.0

11. f(x) = x + 3, g(x) = -x + 2. Find (f + g)(x)

Answers

Answer:

(f + g)(x) = 5

General Formulas and Concepts:

Pre-Algebra

Order of Operations: BPEMDAS

Brackets Parenthesis Exponents Multiplication Division Addition Subtraction Left to Right

Algebra I

Terms/CoefficientsFunctionsFunction Notation

Step-by-step explanation:

Step 1: Define

Identify

f(x) = x + 3

g(x) = -x + 2

(f + g)(x) is f(x) + g(x)

Step 2: Find

Substitute in function values:                                                                          (f + g)(x) = x + 3 - x + 2[Subtraction] Combine like terms (x):                                                              (f + g)(x) = 3 + 2Add:                                                                                                                   (f + g)(x) = 5

I need the area please

Answers

Answer:

120

Step-by-step explanation:

Figure shown is a parallelogram

Area of a parallelogram = base length times height

Given base length: 15in

Given height:8in

Area = 8 * 15 = 120in^2

Answer:

120 in.²

Step-by-step explanation:

8 in. × 15 in. = 120 in.²

In general, half of all the customers you speak to will ask about any upcoming pricing discounts or promotions. Of those who do ask, only 20% qualify for the discount. So that means if I speak to 300 customers, how many will qualify for the discount

Answers

Answer:60

Step-by-step explanation:

300 of 20% is 60

The number of qualified customers for the discount will be 60.

What is the percentage?

The Percentage is defined as representing any number with respect to 100. It is denoted by the sign %. The percentage stands for "out of 100." Imagine any measurement or object being divided into 100 equal bits.

It is given that half of all the customers you speak to will ask about any upcoming pricing discounts or promotions. Of those who do ask, only 20% qualify for the discount. So that means if I speak to 300 customers.

The number of qualified customers will be calculated as below:-

The qualified customers are 20% of 300.

Number = 300 x (20/100)

Number = 300 x 0.2

Multiply the number 330 with 0.2 to get the final value.

Number = 60

Therefore, the number of qualified customers for the discount will be 60.

To know more about percentages follow

https://brainly.com/question/24304697

#SPJ2

Solve the compound inequality

Answers

Answer:

B

Step-by-step explanation:

Answer:

I'm  so sorry I did not mean to answer! Ignore my response

Step-by-step explanation:

Riley's cat has 8 kittens. Of those 8 kittens, 4 are brown, 1 is grey, and the rest are orange. Which of the following could be used to find the probability that the first kitten Riley picks up at random is NOT orange?

Answers

Answer:

5/8 or 62.5 percent

Step-by-step explanation:

Other Questions
The sum of two numbers is 106. The bigger number is 22 more than the smaller number. Then what is half of the smaller number?(1 Point)a) 84b) 21c) 42d) 327Rakesh's mother's age is three times the present age of Rakesh. After 5 years their ages when added together, will become 66 years. The equation that represents the above is as follows;(1 Point)a) x + 3x = 66b) x + 3x + 5 = 66c) 4x + 15 = 66d) 4x + 10 = 668Equation 1 is : 2 x - 3 = - 9 and Equation 2 is : x - 7 = 2. The first equation has solution (x = - 3), and the second equation has solution (x = 9) and both these answers are correct for the respective equations. Choose the correct statement from below.(1 Point)a) Since the first equation has a negative solution, the second equation only is a Linear equation, as it has a positive solutionb) Both equations have solutions which are rational numbersc) The first equation is called a Negative Linear Equation as it has a negative solutiond) None of the statements a, b or c are True9Please see the Figure A, and choose the correct statement that matches the figure from the options given below.(1 Point)a) The above figure is a Polygon with 5 sidesb) The above figure is a Concave Polygonc) The above figure is not a Polygond) The above figure is a Regular Pentagon10The sum of measures of the exterior angles of any polygon is (1 Point)a) 180 degreesb) 360 degreesc) ( n 2 ) x 180 degreesd) ( n 2 ) x 360 degrees11In a quadrilateral, if all sides are equal and the diagonals bisect each other at right angles, then the quadrilateral (1 Point)a) Is a Squareb) Is a Rhombusc) Could be a Square, or could be a Rhombus, we need to know more about the length of the diagonalsd) None of the above statements is correct12A rectangular wire frame could be bent and turned into a Parallelogram keeping the same 4 original sides, but the following must be true for this to happen..(1 Point)a) That the diagonals remain equalb) That the diagonals continue to bisect the angles at the vertices equallyc) The lengths of the diagonals changed) The diagonals are now bisecting each other in 4 equal parts13Which statement is True for the following two Figures A and B, given below(1 Point)a) Figure A is a Rhombus and Figure B is a Kiteb) Figure A and B are both Rhombusc) Figure A and B are both Kitesd) Figure A is a Kite and Figure B is a Rhombus14Which statement out of the four options below, is True?(1 Point)a) All Rectangles are Squaresb) All Rhombuses are Parallelogramsc) All Squares are Concave Polygonsd) All Convex Polygons have diagonals outside them15(1 Point)16If 4368 28 = 156, then if the same 4368 is now divided by 0.28, what will be the answer?(1 Point)a) 156b) 15600c) 15.6d) 1.561712.00001 0.600001 is closest to ..(1 Point)a) 0.02b) 2c) 20d) 20018Complete:(1 Point)19The equation : 12 x + 16 = 20 is the same as (1 Point)a) 12 x + 8 = 12b) 12 x = 20 + 16c) 12 x 16 = 20 16d) 12 x 20 = 162020. Find the value of a quarter of the angle QRT which is x if angle PRS is 2x as shown in the figure below. Consider Triangle PRQ and Triangle RST to be equilateral triangles of different sizes.(1 Point)21Which number will give us a quotient of - 4, if we divide - 24 by ..(1 Point)a) 4b) - 6c) - 4d) 622If 2 ( 3 2 x ) = 3 ( x 5 ) then x = .(1 Point)a) 3b) 3c) 6d) 623The teacher asked the class to divide a given number by 24- Rohit divided the number by 24 and got the correct answer 20- Mohit divided the number by 6 and got an answer which he again divided by 4 and obtained the final answer- Rakesh divided the number by 2.4 and then divided the answer again by 10- Allen divided the number by 2.4 and then multiplied the answer obtained by 10 Now choose the best solution from the below options.(1 Point)a) Only Rohit got the correct answerb) Both Rohit and Mohit got the correct answerc) Rohit, Mohit and Rakesh all got the correct answerd) Rohit, Mohit and Allen all got the correct answer24Abhay and Binu both started walking starting at the same time and at the same rate of 5 km / hour. Abhay walked on the boundary of a big circle and Binu walked on the boundary of a small circle and both these circles were having the same center. Abhay completed 3 circles, whereas Binu completed 7 circles after half an hour. After half an hour ...................(1 Point)a) Abhay has walked more than Binu, as he walked on the boundary of the bigger circleb) Binu has walked more than Abhay, as he has walked more circles than Abhayc) Both of them have covered the same distanced) With the information given above, we cannot say who walked more25Which sign will you put in between [ 120 .. ( 3 / 11 ) ], so that the answer or result is the highest number(1 Point)a) +b) c) xd) Shelton Company has the following account balances at year-end: Accounts receivable $140,000 Allowance for doubtful accounts 7,200 Sales discounts 4,800 Shelton should report accounts receivable at a net amount of:__________a. $120,000. b. $ 12,800.c. $1 08,000. d. $115,200 What percent of 8 is 1,280? A 16096 O B. 16,000% O O O C. 1,600% D. 1.6% does anyone know how to solve this Does this graph represent a function? Why or why not?10008NJ2-10 -8 6 4-2,2 488 10-8--10+O A. No, because it is not a straight line.O B. Yes, because it is a curved line. C. No, because it fails the vertical line test.D. Yes, because it passes the vertical line test. There are 78 square meters And 13 meters I need to find the width Which most contributed to the Battle of Stalingrad becoming a turning point in the war?The battle marked the beginning of Germanys victory.The German army exhausted all of their supplies.The battle marked the end of Germanys advancement.The German army gained hundreds of additional soldiers. How does radon move up through rock and soil? Javier computer services began operations in July 2017. At the end of the company prepares monthly financial statements. It has the following information for the month.a. At July 31, the company owed employees $1,800 in salaries that the company will pay in August. b. On July 1, the company borrowed $40,000 from a local bank on a 10-year note. The annual interest rate is 12%. c. Service revenue unrecorded in July totaled $3,000.Required:Prepare the adjusting entries needed at July 31, 2017. which expression could be used to determine the product of -4 and 3 1/4 All of the following are examples of post secondary education except A company sold 3,000 computers in one month, but 12 were returned. If 3,500 were sold the next month, the company would expect 14 to be returned.What is a valid proportion to represent the problem? Question 1If mZHOE =19 and mZHOP=48, find mZEOP.HO Hannahs favourite cereal comes in 4 sizes. Which is the better buy? (Hint: check each one) *A, 200 g for $1.58B, 400 g for $2.97C, 775 g for $5.49D, 1000 g for $7.52 Help if you don't know, don't touch it [tex]x {}^{2} - 4x + 58 = 23[/tex][tex] {4}^{2} \times 2 - 56 + 2 \sqrt{4} [/tex]please,help me with these equations A _____ is a model of an atom in which each dot represents a valence electron Monetarists believe that changes in the supply of money Question 24 options: do not affect aggregate demand. affect aggregate demand through the loanable funds market only. affect only the investment component of aggregate demand. affect aggregate demand directly. tell whether the given value is a solution of the inequality. please answer these questions i will mark you brainliest if p=2i+4j+3k and q=I+5j-2k,find Pq.